Find the missing factor.15 x ____ = 3.6A.0.24B.0.36C.24D.36

Answers

Answer 1

Answer:

24

Step-by-step explanation:

Solving with decimals

.15 * ? = 3.6

Divide each side by .15

? = 3.6/.15

Multiply the top and bottom by 100

? = 360/15

? = 24


Related Questions

Can you please help me


What is the solution to -1-7?
H
+
-10-9-8-7-6-5-4-3-2-1 0 1 2 3 4 5 6 7 8 9 10
0-8
O-6
06
08

Answers

Answer:

Please be more specific

Step-by-step explanation:

The graph shows the amount of savings over time in Eliana​'s account. Lana​, ​meanwhile, puts ​$45 each week into her savings account. If they both begin with​ $0, who is saving at the greater​ rate?

Answers

Answer:

first you didn't tell us how much money eliana saves. you just said about Lana's saving.

What type of connection can you make between markups and tips or taxes? What type of connection can you make between markdowns and discounts? Explain your answer.

Answers

Markup adds an increase in the original price of an item, and tips or taxes add money to a bill. Hence, the connection between markups and tips or taxes.

Discounts are given in order to lower the price of an item from its original price, and markdown is the price that is reduced from an item's original price.

What is the difference between Markup and Markdown?
Markup price is the increase in an item (by a certain percentage) over the original price.

Markdown prices are reductions in an item's selling price from its initial selling price expressed as a rate (markdown percentage).

To learn more about Markup and Markdown, use the link given below
https://brainly.com/question/3099549
#SPJ1

f i deposit $100 each month into an account earning 8% interest compounded monthly, how much will i have in the account in 35 years?

Answers

The amount of money in the account in 35 years to the nearest cent is $1629.25.

What is the accrued amount in 35 years?

The compound interest formula is expressed as;

A = P(1 + r/n)^(n*t)

Where A is accrued amount, P is principal, r is rate,  and t is time.

Given that;

Principal P = $100Interest rate r = 8% = 8/100 = 0.08Compounded monthly n = 12 Time t = 35 yearsAccrued amount A = ?

Plug the given values into the compound interest formula and solve for A.

A = P(1 + r/n)^(n*t)

A = $100( 1 + 0.08/12)^( 12 × 35)

A = $100( 1 + 0.08/12)⁴²⁰

A = $1629.25

Therefore, the accrued amount is $1,629.25.

Learn more about compound interest here: brainly.com/question/27128740

#SPJ1

29. Find x and the dimensions of the rectangle below.
A = 126 ft²
x+2
x-3

Answers

The x value is 12, length is 14 feet and breath is 9 feet when the area of rectangle is 126 square feet.

Given that,

The rectangle has a surface area of 126 square feet.

The length is x+2 and the breath is x-3.

We have to find the x value, length and breath.

We know that,

The area of the rectangle is length × breath.

126=(x+2)(x-3)

126= x²-3x+2x-6

126= x²-x-6

x²-x-6-126=0

x²-x-132=0

x²-12x+11x-132=0

x(x-12)+11(x-12)=0

(x-12)(x+11)=0

x-12=0     or   x+11=0

x=12        or    x=-11

We take the x as positive so its 12.

Length is x+2=12+2=14

Breath is x-3=12-3=9

Therefore, The x value is 12, length is 14 feet and breath is 9 feet when the area of rectangle is 126 square feet.

To learn more about rectangle visit: https://brainly.com/question/15019502

#SPJ1

The population of a city has increased by 24% since it was last measured. If the current population is 65,100, what was the previous population?

Answers

Answer: 49476

Step-by-step explanation:

trying to find the logarithm

Answers

The expression of logarithm will be -In4 x k .

Given,

In the question:

The expression of the logarithms is:

In[tex](\frac{1}{4^k} )[/tex]

To solve the above expression:

Now, According to the question:

In([tex]\frac{1}{4^k}[/tex])

Apply laws of logarithm to simplify the expression

-In[tex]4^k[/tex]

Express the logarithm of a power of an expression as the power times the logarithm of the expression

-k x In4

Multiply the monomials :

-In 4 x k

Hence, The expression of logarithm will be -In4 x k .

Learn more about Logarithm at:

https://brainly.com/question/28346542

#SPJ1

IXL Last One ! for this assignment! Thank you guys!

Answers

Answer:

If not mistaken the point of X is (37 -5) toward the middle because that is where it could be that I can see.

Step-by-step explanation:

Answer:

x=5

Step-by-step explanation:

59-2x=64-3x

59+x=64

x=5

Hopes this helps please mark brainliest

A box contains 17 calculators, 5 of which are defective.
If five calculators are randomly selected from the box, in how many ways can all the selected calculators be defective?

There are _____ different ways for all 5 selected calculators to be defective.

If five calculators are randomly selected from the box, in how many ways can all the selected calculators be non-defective?

There are _____ different ways to for all 5 selected calculators to be non-defective.

Answers

There is only 1 way for all 5 selected calculators to be defective.

There are 792 different ways for all 5 selected calculators to be non-defective.

Given, that a box contains 17 calculators, 5 of which are defective.

So, there are 12 non-defective calculators.

Now, five calculators are selected randomly from the box.

We are asked the number of ways in which all the selected calculators are defective.

So, the number of ways in which all selected 5 calculators be defective is,

C(5 , 5) i.e. we only select 5 defective calculators.

C(5 , 5) = 1.

So, there is only 1 way for all 5 selected calculators to be defective.

Now, we are asked the number of ways in which all the selected calculators are non-defective.

So, the number of ways in which all selected 5 calculators be non-defective is,

C(12 , 5) i.e. we select 5 calculators from the 12 non-defective calculators.

C(12 , 5) = (12×11×10×9×8)/(5×4×3×2×1)

C(12 , 5) = 792

So, there are 792 different ways for all 5 selected calculators to be non-defective.

Hence, there is only 1 way for all 5 selected calculators to be defective.

There are 792 different ways for all 5 selected calculators to be non-defective.

Learn more about Permutations and Combinations here https://brainly.com/question/1216161

#SPJ1

Plsssssssss helpppppppppppp

Answers

Answer: It is C

Step-by-step explanation:

The slop for Kitten A is 1/4 or 4

Kitten B has a slop of 2/6 or 3 so the answer is,

C Kitten B/rate of change 4

What is the approximate percent decrease from 1865 to 1240?

Answers

Answer: It should be 33.5%

1.) Solve the following equation for w: 3w-4(w-3) = 6

Answers

Answer: w=6

Step-by-step explanation: 1. Simplify. Distribute the -4 to the w and the -3. -4*w is -4w and -4*-3 is +12, so your left with 3w-4w+12=6.


2. simplify again. 3w-4w is -w.

3. -w+12=6. Subtract w to both sides, so its -w=-6. The negatives cancel out, so w just equals 6.

In a baseball league of six teams, how many games are needed to complete the schedule if each team plays seven games with each other team?

Answers

If the six teams of a baseball league , and each team plays seven games with each other , then the number of games to be played is 105  .

in the question ,

it is given that

the number of teams in the baseball league = 6

number of matches that each team plays with each other = 7 .

Number of ways two teams can be selected out of 6 teams = C(6,2)

and they need to play 7 games with each other ,

so the number of games required to complete the schedule is

= 7 × C(6,2)

= 7 × 15

= 105

Therefore , the number of matches that is needed to complete the schedule is 105 .

Learn more about Combination here

https://brainly.com/question/28472805

#SPJ1

a8=12, a16=22 Write a rule for the nth term of the arithmetic sequence.

Answers

The formula for the arithmetic sequence that contains a(8) = 12 and a(16) = 22 is a(n) = 13 / 4 + (5 / 4) · (n - 1).

How to derive the formula for the arithmetic series

In this problem we must derive the formula behind the arithmetic series. Arithmetic series are sets of numbers generated by expression of the form:

a(n) = a' + r · (n - 1)

Where:

a' - Value of the first element of the series.r - Difference between two consecutive elements.n - Index of the n-th element.

If we know that a(8) = 12 and a(16) = 22, then the formula for the arithmetic formula is:

12 = a' + 7 · r

22 = a' + 15 · r

Now we find a system of two linear equations with two variables, whose solution is found by using numerical methods:

(a', r) = (13 / 4, 5 / 4)

Then, the formula for the arithmetic sequence is a(n) = 13 / 4 + (5 / 4) · (n - 1).

To learn more on arithmetic sequences: https://brainly.com/question/10396151

#SPJ1

hey i need help asap with this homework problem.

Answers

The amount in Mrs. Piazza's account after 4 years is 8960 dollars.

What is simple interest?

Simple interest is a method to calculate the amount of interest charged on a sum at a given rate and for a given period of time. In simple interest, the principal amount is always the same, unlike compound interest where we add the interest of previous years principal to calculate the interest of the next year.

Principal(P) = 8000

time(T) = 4 years

rate(R) = 3%

simple interest = PRT/100

Simple interest = 8000 x 3 x 4 / 100

simple interest is 960 dollars

Amount after 4 years = principal + interest

                                    = 8000 + 960

                                    = 8960 dollars

In conclusion, the Amount after 4 years is 8960 dollars

Learn more about simple interest: https://brainly.com/question/25793394

#SPJ1

Can someone help me I don’t understand this question it’s geometry

Answers

The triangle with all sides is are given below

What is a triangle?

Triangles are three-sided closed polygon formed by the intersection of three lines. It is encountered a lot in everyday life. It is one of the basic shapes of geometry. It has three sides, three angles, and three vertices.

Given, angle A =30, b=6

we know cos 30 = √3/2

√3/2 = b/c

√3/2=6/c

c =6/√3/2= 6.9

Similarly for a=13, b =13

using pythagoras theorem,

c^2=a^2+b^2

=18.38

Hence, we can write the rest part

To know more about pythagoras theorem, visit:

https://brainly.com/question/11070154

#SPJ1

what is equivalent to (5+i)+(7-4i)
a. 2-3i
b. 2-5i
c. 12-3i
d. 12-5i

Answers

The answer is c. 12-3i you find it by adding 5&7 and i&-4i which gives you 12-3i

Work out 132% of 96 m
Give your answer as a decimal in metres (m).

Answers

Answer:

126.72m

Step-by-step explanation: 1.32 x 96 = 126.72, I struggled with that stuff too no shame

Find the perimeter or circumference and area of each figure if each unit on the graph measures

Answers

The circumference of the given circle is 17.8 cm and the area of the given circle is 25.1 [tex]cm^2[/tex]

What is area and circumference of circle?

Circumference of circle is the length of the boundary of the circle

If the radius of the circle is r, then the circumference of the circle is [tex]2\pi r[/tex]

Area of the circle is the total space taken by the circle

If the radius of the circle is r, then the area of the circle is [tex]\pi r^2[/tex]

Center of the circle = (2, 3)

Point on a circle = (4, 1 )

Radius of the circle =

[tex]\sqrt{(4-2)^2 + (1 - 3)^2}\\\sqrt{2^2 + (-2)^2}\\\sqrt{4 + 4}\\\sqrt{8}[/tex]

Circumference of circle = [tex]2 \pi \sqrt{8}[/tex]

                                       = [tex]2 \times 3.14 \times 2 \times \sqrt{2}[/tex]

                                       = [tex]2 \times 3.14 \times 2 \times 1.414[/tex]

                                       = 17.8 cm

Area of circle = [tex]\pi \times \sqrt{8} \times \sqrt{8}\\[/tex]

                       = [tex]3.14 \times 8[/tex]

                       = 25.1 [tex]cm^2[/tex]

To learn more about area and circumference of circle, refer to the link-

https://brainly.com/question/13644101

#SPJ9

Complete Question

The figure has been attached here

Find the perimeter or circumference and area of each figure if each unit on the graph measures 1 cm. Round answers to the nearest tenth, if necessary.

Which has greatest rate help please

Answers

The last function having the greatest rate of change.

What is rate of change?

The pace at which one quantity changes in relation to another quantity is known as the rate of change function. Simply said, the rate of change is calculated by dividing the amount of change in one item by the equal amount of change in another.

Consider the first function,

From this function we get two points (2, 1) and (-2, -1).

To find rate using this points.

Rate = [tex]\frac{y_2-y_1}{x_2-x_1} = \frac{-1-1}{-2-2} = \frac{-2}{-4}=\frac{2}{4} =\frac{1}{2}[/tex]

Consider the second function,

The second function is a vertical line of x = -3.

Since the rate of vertical line is undefined.

Consider the third function,

The third function is a horizontal line y =  4.

Since two points having the same y - value.

Hence, the rate is zero.

Consider, the last function,

From last function we get two points (1, 5) and (-3, -3).

So,

Rate = [tex]\frac{-3-5}{-3-1} = \frac{-8}{-4} = \frac{8}{4} = 2[/tex]

Hence, the last function having the greatest rate.

To know more about the rate of change, click on the link

https://brainly.com/question/8728504

#SPJ1

Suppose an individual makes an initial investment of $2,800 in an account that earns 6%, compounded monthly, and makes additional contributions of $100 at the end of each month for a period of 12 years. After these 12 years, this individual wants to make withdrawals at the end of each month for the next 5 years (so that the account balance will be reduced to $0). (Round your answers to the nearest cent.)
(a) How much is in the account after the last deposit is made?
$



(b) How much was deposited?
$



(c) What is the amount of each withdrawal?
$

(d) What is the total amount withdrawn?
$

Answers

a) The total amount in the account after the last deposit is $26755.96.

b) A total of $16900 was deposited.

c) Amount of each withdrawal is $517.

d) The total amount withdrawn is $31020.

What is future value?

The worth of a present asset at some point in the future based on an estimated rate of growth is known as future value (FV). For investors and financial planners, the future value is crucial because they use it to predict how much an investment made now will be worth in the future.

FV of a single deposit of $ 2800 can be computed as

FV₁ = PV(1+r)ⁿ

PV = $ 2800

r = 6% = 0.06/12

r = 0.005

n= number of periods

= 12 yr × 12 months = 144

FV₁ = 2800 (1 + 0.005)¹⁴⁴

= 2800 (1.005)¹⁴⁴

= 2800(2.0507)

= 5741.96

The FV of monthly deposits can be computed using the formula for the FV of the annuity:

FV₂ = C x [(1+r) n -1/r]

Periodic cash deposit, C = $100

So,

FV = 100 × [((1.055)¹⁴⁴ - 1)/0.005]

= 100 × [(2.0507 - 1)/0.005]

= 100 × [1.0507/0.005]

= 100 × 210.14

= $21014

The total future value of the account is given by

= $5741.96 + $21014

= $26755.96

b) Total deposit = Initial investment amount + amount of monthly deposits

= 2800 + (100 × 144)

= 2800 +14400

= 16900

c) Formula for PV of an annuity can be used to compute monthly withdrawals as:

PV = A x [1-(1+r)⁻ⁿ/r]

So, A = PV/[1-(1+r)⁻ⁿ/r]

PV = $26755.96

n= number of periods = 5 × 12 = 60

r = 0.005 per month

A = 26755.96/ [1- (1+0.005)⁻⁶⁰/0.005]

A = 26755.96/ [1- (1.005)⁻⁶⁰/0.005]

A = 26755.96/ [1- (0.7413)/0.005]

= 26755.96/ [0.2587/0.005]

= 26755.96/(51.74)

= 517.12

So, the amount of each withdrawal for five years is $517.

d) Total amount withdrawn is

= number of withdrawals × amount of each withdrawal

= 60 × 517

=31020

To know more about future value, visit:

https://brainly.com/question/14115040

#SPJ13

If Elon and Jeff work together, they can finish the work in 3 hours. If Jeff works together with Bruce, they
can make this work done in 6 hours. Elon and Bruce can finish this work in 4 hours. How much time would
it take to finish the work if all three worked together? pls helpp

Answers

Answer:

  2 2/3 hours

Step-by-step explanation:

You want to know the time it would take Elon, Jeff, and Bruce working together, if Elon and Jeff can complete the work in 3 hours, Jeff and Bruce take 6 hours, and Elon and Bruce take 4 hours.

Total of rates

Job completion time problems like this one are more easily understood in terms of rate of completion. If we let e, j, and b represent the rates of job completion in jobs per hour for each of Elon, Jeff, and Bruce, respectively, then the given relations can be written ...

  e +j = 1/3

  j +b = 1/6

  e +b = 1/4

Adding these equations together gives ...

  (e +j) +(j +b) +(e +b) = (1/3) +(1/6) +(1/4)

  2(e +j +b) = 3/4 . . . . . . . simplify

  e + j + b = 3/8 . . . . . . . . divide by 2

The total rate of job completion when Elon, Jeff, and Bruce work together is 3/8 job per hour. The hours it takes them to do one job is the reciprocal of this:

  1/(3/8 job/hour) = 8/3 hour/job

It would take Elon, Jeff, and Bruce 2 2/3 hours to complete the work if they worked together.

__

Additional comment

For this problem, we don't need to know the individual completion times. If we were to figure them we'd find them to be (working alone) ...

Elon: 4.8 hoursJeff: 8 hoursBruce: 24 hours

Suppose you want to have $500,000 for retirement in 35 years. Your account earns 4% interest. How much would you need to deposit in the account each month?

Answers

The answer, the money needs to be deposited each month is $557.47.

What is simple interest?

A quick and simple way to figure out interest on money is to use the simple interest technique, which adds interest at the same rate for each time cycle and always to the initial principal amount. Any bank where we deposit our funds will pay us interest on our investment. One of the different types of interest charged by banks is simple interest. Now, before exploring the idea of basic curiosity in further detail,

The future money plan to have 500000

Retirement in 35 years.

The interest 4%.

number of time periods = 35 years * 12 months per year = 420

interest will be .4166666667% per month

The formula = [tex]p(\frac{(1+r/n)^{r/n} -1 }{\frac{r}{n} } )[/tex]

⇒500000*(.014/12) = d(3.98-1)

⇒d = 1666.7/2.98 = 557.47

Hence the money needs to be deposited each month is $557.47

Learn more about simple interest, by the following link

brainly.com/question/20690803

#SPJ1

I need help with this

Answers

The value of b is 405 in the given polynomial (a+3)⁵=a⁵+15a⁴+90a³3²+270a²+ba+243

What is Polynomial?

Polynomial is an expression consisting of indeterminates and coefficients, that involves only the operations of addition, subtraction, multiplication, and positive-integer powers of variables.

Given,

(a+3)⁵=a⁵+15a⁴+90a³3²+270a²+ba+243,

We need to find the value of b

The binomial formula for (a+b)⁵=a⁵+5a⁴b+10a³b²+10a²b³+5ab⁴+b⁵

Now let us find (a+3)⁵

We just need to replace b with 3 in above formula.

(a+3)⁵=a⁵+5a⁴3+10a³3²+10a²3³+5a3⁴+3⁵

=a⁵+15a⁴+90a³3²+270a²+405a+243

Hence the value of b is 405 in the given polynomial a⁵+5a⁴b+10a³b²+10a²b³+5ab⁴+b⁵

To learn more on Polynomials click:

https://brainly.com/question/11536910

#SPJ1

$8 is what percent of $8?

Answers

Answer:

100%?

Step-by-step explanation:

DUE TOMORROW :)) PLEASE HELPPP :))

Answers

Answer:

x = 10

Step-by-step explanation:

The angles are supplemental

100 -2x + 10x = 180  Combine like terms

100 + 8x = 180  Subtract 100 from both sides of the equation

8x = 80  Divide both sides by 8

x = 10

You deposit $500 in an account earning 4% interest compounded annually. How much will you have
in the account in 15 years?

Answers

The total amount  will have in the account in 15 years is $900.47

What is compound interest?

Compound Interest is the interest calculated on the cumulative amount, rather than being calculated on the principal amount only.

Amount,[tex]A=P[1+(\frac{R}{100})^{T}][/tex], where P is the principal, R is the rate of interest per unit time period and n is the time period.

Compound Interest, CI = Amount – Principal

Using the formula,

                     [tex]A=P[1+(\frac{R}{100})^{T}][/tex]

Where,

             A=Total Amount(To be calculated)

             P=Principal of money deposited=$500

             R=Annual interest rate=4%

             T=Time in years=15 years

⇒[tex]A=500(1+\frac{4}{100}[/tex][tex])^{15}[/tex]

⇒A=500(1+0.04[tex])^{15}[/tex]

⇒A=500(1.04[tex])^{15}[/tex]

⇒A=500*1.80094351

⇒A=$900.47

Hence,the total amount will have in account in 15 years is $900.47

To learn more about compound interest,visit:

https://brainly.com/question/29370378

#SPJ9

2.64 to nearest whole number

Answers

3.
.6 would round up to one and one plus 2 is 3.

Answer:

3

Step-by-step explanation:

2.64 rounded to the nearest whole number would be 3. This is because if a number is 5 or greater than 5, it rounds up, but if a number is under 5, it would round down. Since we are looking at the number to the right of the 2 in 2.64, and because we are looking for the nearest whole number, it would round up to 3. 6 is greater than 5, so we automatically know that we round up.

Given that a function, g, has a domain of -20 ≤ x ≤ 5 and a range of -5 ≤ g(x) ≤ 45 and that g(0) = -2 and g(-9) = 6, select the statement that could be true for g.

Answers

The correct option for the given function is (D) g(-13) = 20.

Given that:

The function g has domain -20 ≤ x ≤ 5, range -5 ≤ g(x) ≤ 45, g(0) = -2, g(-9) = 6.

Let's analyze each option to see if it fits the given domain.

A. g(7) = -1

At the beginning of the problem, it is said that the domain is restricted to the range [-20, 5]. The input x = 7 here is outside this domain. That is, it cannot be true for function g without being in the domain. So g(7) is not possible.

B. g(-4) = -11

Similarly, it is said that the domain is restricted to the range [-20, 5]. The input x = -4 here is outside this domain. That is, it cannot be true for function g without being in the domain.

C. g(0) = 2

We know in the question that g(0) = -2. So, this is also the wrong option.

D. g(-13) = 20

Unlike the other options, x= -13 is in the range of -20 to 5. g(-13) = 20 is also in the range of -5 to 45. Option D is true for a function g.

x= -13 is in range and 20 is also in range. So this applies to g.

From the statement given,

g(-13) = 20 is true for the value of g.

To learn more about the function visit: https://brainly.com/question/5975436

#SPJ1

Which is the graph of y=LxJ-2?

Answers

Answer:

the third one is the answer.

Step-by-step explanation:

take 0 at the x value, the greatest integer function of 0 is 0 its self. so when you take 0, Y=[x]-2 become -2. so the y-axis Crosses in (0,-2).

Other Questions
In the balanced equation, what is the coefficient for the reactant Nb?Nb + O2 point 2Nb2O5 Electrolytes help control fluid balance via __________, the movement of water through a semipermeable membrane based on solute concentration. For rectangles with the same area, the length varies inversely to the width. Henry is trying to decide how to plan the dimensions of his new rectangular swimming pool. He is planning on his pool being 200 square feet. One option would be to have a length of 20 ft and a width of 10ft. Write an equation to model this scenario.PLEASE HELP WILLMARK BRAINLIESTA: 10=a/20B: 20=a/10C: 10=20/aD: 20= 10/a Write meaningful sentences using the given signal words:For exampleIndeedIn factIn other wordsNamelyPLEASE HELP ME ASAP I NEED THIS HURRY PLEASE!!!If you change the difference in lag times of P and S waves from 00:01:50 to 00:04:00,how does that change the distance of the earthquake to the seismic station? intermittent production processes are used to produce a large variety of products with different processing requirements in lower volumes. intermittent production processes are used to produce a large variety of products with different processing requirements in lower volumes. true false the doctrine of swine objection that mill considers against hedonic utilitarianism is an objection that While typing your PIN number into the banking app on your smartphone, you notice some creepy guy trying to peek at what you're doing. what was the main reson that the belgian colonial government in rwanada established a national identity card that included the category ethinicyt sixteen-year-old jake is trying out different clothes and hairstyles. his father is confused and sometimes shocked by jake's combinations of shirts and pants, earrings, chains, and hair colors. jake's mother, on the other hand, just laughs. jake is in the stage of development called: Bayview apartments is a 750-unit apartment complex. when the apartments are 90% occupied, monthly operating costs total $213,525. when occupancy dips to 80%, monthly operating costs fall to $208,800. the owner of the apartment complex is worried because many of the apartment residents work at a nearby manufacturing plant that has just announced it will close in three months. the apartment owner fears that occupancy of her apartments will drop to 65% if residents lose their jobs and move away. assuming the same relevant range, what can the owner expect her operating costs to be if occupancy falls to 65%? If I have a book that is 298 pages, and I read 20 pages a day, how long will it take me to read the entire book? Also what is the formula to solve this problem problem!! Ok, I found out how to find complementary angles. But now I learned out how find supplementary angles. I have homework that is due tomorrow and I forgot my notes from today, someone please help me find this angle asap! 2. .a) (6 + a) * 7;6) 5 (10-x) What is the general context of the poem new Nepal How did white reactions to slave resistance affect democratic society in the South? How does cellular respiration differ from photosynthesis?Cellular respiration uses glucose as the main source of energy, while photosynthesis uses light/radiant energy as the main source of energy.The final product of cellular respiration is electromagnetic waves, while the final product of photosynthesis is oxygen.Cellular respiration uses energy stored in proteins, while photosynthesis uses energy stored in lipids.Cellular respiration uses water as the main source of energy, while photosynthesis uses energy stored in carbohydrates. 6, what is the standard deviation of the residual values in the regression output. note that the residual values measure the difference between the observed values of the travel time in hours and the predicted values of the travel time in hours from the regression model. Describe the 2 main steps in protein synthesis, along with the key molecules involved in the process. infants will stare at an object with repeated presentations of that object. this demonstrates the psychological principle of .